A water bottle holds 7/8 of a gallon of water. There are 5 1/4 gallons of water in a water tank. How many water bottles can be filled with the water in the tank?

Answers

Answer 1

Answer:

6 water bottles


Related Questions

Help its timed 35 points

Answers

Answer:
x = 36
Angle 1 = 108
Angle 2 = 72

Explanation: Angle 1 & Angle 2 are supplementary, equaling to 180 You can just add them both and then equal to 180.

3x + 72 = 180

Out of 90 scientists who study dinosaurs, 30 study the T-Rex, 22 study the Brontosaurs and 10 study both. How many of this group of scientists studies another speices of dinosaurs?

Answers

Answer:

it 152 u have to add 3 time like this

90+10+30+ it gives you 152

Step-by-step explanation:

hope this works or help

How much water does it take to completely fill a pool that is 50 m long, 25 m wide, and 2.5 m deep?

:)

Answers

Answer: 3125m

Step-by-step explanation:

volume = h · d · w

V= 2.5 * 50 * 25

v= 3125

Eric made a scale drawing of the auditorium. The stage, which 15 35 feet wide in real life,
Is 5 ipches wide in the drawing, What is the scale of the drawing?
by

Answers

Based on the given conditions, formulate: 5/35
Simplify by dividing by dividing the numerator and denominator by 5: 1/7

Therefore the scale of the drawing is 1/7

come get a brainliest

Answers

Answer:

the answer of first one is19

the answer of second is14

A salsa recipe uses green pepper, onion, and tomato in the extended ratio 1 : 5 : 7. How many cups of onion are needed to make 117 cups of salsa?

Answers

Answer:

We need 45 cups on onion

Step-by-step explanation:

green pepper : onion: tomato: total

1                          5              7        13

We have 117 total so divide by 13

117/13 = 9

Multiply each number by 9

green pepper : onion: tomato: total

1*9                        5*9        7*9        13*9

9                            45         63           117

We need 45 cups on onion

Bryan brought his series set of Rowan and the Dragon Quest to a secondhand store to sell. He was paid cash for all 5 books in the set. Before he left, Bryan used $29.75 of his earnings to purchase a used e-reader. He had $2.75 remaining. What was the value of each book in the set?

Answers

The value of each book in the set will be $6.5 when he sold his books to the second hand store.

What is profit and loss?

When we sell any product above its cost price then there will be profit if the selling price is less then the cost price then there will be loss on the product.

here the total cost of the books sold will be given as:-

[tex]\$29.75+ \$2.75 =\$ 32.5[/tex]

The cost of the each book will be

[tex]=\dfrac{32.5}{5}=\$6.5[/tex]

Hence the value of each book in the set will be $6.5 when he sold his books to the second hand store.

To know more about Profit and Loss follow

https://brainly.com/question/19104371

#SPJ1

Answer:

$6.50

Step-by-step explanation:

Use the data set to answer the question. PLS NEED HELP!! No files or links please! :))

{122,132,155,198,213,225,261,280}

What is the mean absolute deviation for the data set?

Enter your answer as a number rounded to the nearest tenth, like this: 42.5

Answers

[tex]\bold{ANSWER:}[/tex]
46.5

[tex]\bold{EXPLANATIONS:}[/tex]

Can someone solve number 1 only

Answers

Answer:

24.00

Step-by-step explanation:

hope this helps

A store has an item listed for $48.95. On Saturday they are having a 20% off sale where they give a 20% discount to all items in the store. What is the sale price of this item on Saturday? Round appropriately.
(A) None of these
(B) $58.74
(C) $9.79
(D) $28.95
(E) $39.16

Answers

Answer:

(E) $39.16

Step-by-step explanation:

listed price = $48.95

discounted price on saturday = $48.95 x 80% = $39.16

It’s 39.16$ since you divided

Based only on the information given in the diagram, which congruence
theorems or postulates could be given as reasons why A DEF= AJKL?
Check all that apply.
D
O A. LL
B. HL
C. LA
D. ASA
E. SAS
OF. AAS

Answers

The option (C) LA and (D) ASA are correct because two angles and one side are the same.

What is the congruent triangle?

Congruent triangles are those that are exactly the same size and shape. Congruent is represented by the symbol When the three sides and three angles of one triangle match the dimensions of the three sides and three angles of another triangle, they are said to be congruent.

From the triangle DEF and triangle JKL

∠D = ∠J  (given)

∠F = ∠L  (given)

ED = KJ  (given)

∴ Triangle DEF and triangle JKL are congruent by the ASA

ΔDEF≅ΔJKL

Also:

∠F = ∠L  (given)

ED = KJ  (given)

ΔDEF≅ΔJKL by LA

Thus, the option (C) LA and (D) ASA are correct because two angles and one side are the same.

Learn more about the congruent triangle here:

brainly.com/question/12413243

#SPJ1

please help me out with this here is a screen shot

Answers

Corresponding sides of similar triangles are in same proportion.

Is π^2 greater or less than 7.2?

Answers

Answer:

less than

Step-by-step explanation:

Well pi divided by 2 is 1.57 and you need to figure out if 7.2 is greater than or less than pi^2 aka 1.57 so theres your ansewr

Answer:

greater

Step-by-step explanation:

π = 3.1425.....

π > 3

π^2 > 9 > 7.2

What is the mean of this data set?
{10, 5, 7, 9, 5, 18}
Enter your answer in the box.

Answers

             - - - - - - - - - - - - - - - - - - - - - - - - - - - - - - - - - - - - - - - - - - - - - -

[tex]\blue{\textsf{\textbf{\underline{\underline{Question:-}}}}[/tex]

          What is the mean of the dataset (10, 5, 7, 9, 5, 18)?

[tex]\blue{\textsf{\textbf{\underline{\underline{Answer:-}}}}[/tex]

Mean [tex]\rightarrow[/tex] 9

[tex]\blue{\textsf{\textbf{\underline{\underline{How to solve:-}}}}[/tex]

               First, add all the numbers together:-

10+5+7+9+5+18

[tex]\textsf{54}[/tex]

Now divide by how many numbers there are (6):-

54:6=9

So the mean of the dataset is 9.

Good luck.

         - - - - - - - - - - - - - - - - - - - - - - - - - - - - - - - - - - - - - - - - - - - - - - - - - -

Answer:

9

Step-by-step explanation:

To find the mean, add up all the numbers and then divide by the number of terms

(10+5+7+9+5+18)

54

There are 6 terms

54/6=9

The mean is 9

someone help me solve this

Answers

Answer:

Looking at the given points we know that it is going to be rectangle because we have 4 points.  Therefore the figure is a rectangle and the area formula would be [tex]A=l*w[/tex].

You can look at the attached graph and what we see is that our width of the rectangle is from [tex]y=-4[/tex] to [tex]y=3[/tex] which means that our height is 7 units wide.  We can also see that our length starts at [tex]x=-1[/tex] and ends at [tex]x=4[/tex] making us have a length of 5 units.

We then plug in the values and solve

[tex]A = 7\ units * 5\ units[/tex]

[tex]A = 35\ units^2[/tex]

Therefore, the area of our rectangle is [tex]35\ units^2[/tex]

Hope this helps!  Let me know if you have any questions

if you run for 30 mintes at a speed of 8 miles per hour how many feet wpuld you hav erun

Answers

Answer:

not sure if this is correct but i tried (1267200 feet)

Answer:

21120 ft

Step-by-step explanation:

We need to change 8 miles per hour to feet per minute

8 miles      5280 ft      1 hour

----------- * ----------      * ----------

hour          1 mile          60 minutes

704 ft/ minute

Now multiply by 30 minutes

704 ft / minute * 30 minutes =21120 ft

A fast food chain surveyed a random
sample of students to determine what pizza
they prefer. Cheese pizza was preferred
by 40 students. Pepperoni pizza was
preferred by 28 students. Based on the
results, how many of the 340 students can
be expected to prefer cheese pizza?

Answers

Answer:

200 students

Step-by-step explanation:

If 40/68 students prefer cheese pizza, we can take the proportion and carry it over by doing 340/68, which is 5.

5 x 40 = 200.

Three people, Adam, Becky and Charley, each lie 1/3 of the time at random, otherwise they tell the truth. A fair coin is flipped, and they all see it. They all say it is heads. What is the probability the coin was actually tails?

Answers

given:

[tex] \frac{1}{3} [/tex]

to find:

the probability of the coin.

solution:lie:

[tex]( \frac{1}{3} ) \times ( \frac{1}{3}) \times ( \frac{1}{3} )[/tex]

[tex] = \frac{1}{27} [/tex]

there's 1/27 chance of them to lie.

truth:

[tex]( \frac{2}{3} ) \times ( \frac{2}{3} ) \times ( \frac{2}{3} )[/tex]

[tex] = \frac{8}{27} [/tex]

there's 8/27 chance that all of them told the truth.

probability:

[tex]( \frac{8}{27} ) \div (( \frac{8}{27} ) + ( \frac{1}{27} ))[/tex]

[tex] = \frac{8}{9} [/tex]

therefore, between those two chances, it's an 8/9 chance it's actually Heads.

Please help me on this!

Answers

The correct statement is the first one. g(x) exceeds f(x) by 72.

Which statement is true?

By looking at the table, we can see that f(x) is a linear function and g(x) is an exponential function, such that:

f(x) = 10 + 3x

g(x) = (1/2)^x

By evaluating these functions in x = -6 we get:

[tex]f(-6) = 10 - 3*6 = -8[/tex]

[tex]g(-6) = (1/2)^{-6} = 64[/tex]

The difference is:

g(-6) - f(-6) = 64 + 8 = 72

Then the correct option is the first one, g(x) exceeds f(x) by 72.

If you want to learn more about functions:

https://brainly.com/question/4025726

#SPJ1

Three machines produce the same parts.

Machine A produces 10% defective parts.
Machine B produces 5% defective parts.
Machine C produces 5% defective parts.

Suppose each machine produces 1/3 of the total production. A part is selected at random, and it is found to be defective. What is the probability that it was produced by machine B?
A) 19/56
B) 17/30
C) 1/2
D) 1/4

Answers

Answer:

20

Step-by-step explanation:

just add the 13 to A and B

eathan answer 80% correctly on his history test he correctly answers 32 enter the amount of questions on his test

Answers

Answer:

40 questions.

Step-by-step explanation:

By proportion:

80 %      is equivalent to  32 questions

So 100%  ..   ..   ..  ..   ..  ..   (32/80) * 100

= 32 * 5/4

= 160/4

= 40

Answer:

40

Step-by-step explanation:

80/100=32/x

You can also set this up with a diagram.

x=40

Find the volume of a cylindrical tank whose radius is
10cm and the height is 21cm. (Take T=22/4)

Answers

Answer:

Question:

Find the Volume of a Cylindrical Tank whose radius is 10cm and the height is 21cm. (Take π = [tex] {\bold{\frac{22}{7}}}[/tex])

Answer:

We know that,

[tex] \quad [/tex][tex] \quad [/tex][tex] { \boxed{ \sf{Volume \: of \: Cylinder =\pi {r}^{2}h }}}[/tex]

➛ [tex] { \large{\frac{22}{7} \times {(10)}^{2} \times 21}} [/tex]

➛ [tex] { \large{ \frac{22}{7} \times 10 \times 10 \times 21 }} [/tex]

➛ [tex] { \large{ \frac{22}{7} \times 100 \times 21 }} [/tex]

➛ [tex] { \large{ \frac{22}{{ \cancel{7}} ^{1} } \times 100 \times { \cancel{21}}^{3} }} [/tex]

➛ [tex] 22 \times 100 \times 3 [/tex]

➛ [tex] 22 \times 300 [/tex]

➛ [tex] { \boxed{ \green{6600{ \rm{ {cm}^{3}}} }}} [/tex]

Thus, The volume of Cylindrical Tank is[tex]{ \bold{ \: 6600{ { \bold{ {cm}^{3}}} }}}[/tex].

What is the range of f(x)=5/x^4

Answers

Answer:

(0,∞) , {y|y>0}

Step-by-step explanation:

14 ft
16 ft
5 ft
What is the Surface Area??

Answers

Answer:

0.805ft

Step-by-step explanation:

she determines the surface area of material she needs to create

ill give 30 points for the answer bro: In the figure, c || d. What are the measures of ∠1 and ∠2?

Answers

Answer:

angle 1 is 15, angle 2 is 75

Step-by-step explanation:

angles 1/2/3/4 is the same as 5/75 degrees/7/8 respectively

150 books are divided into 3 stacks. The 1st one has 30 more than the 2nd stack, 2nd stack has twice as many books in the 3rd stack. How many books are there in 3rd stack? (N represents the number of books in the 3rd stack.)

Answers

Answer:  24 books

======================================================

Explanation:

N = number of books in the 3rd stack2N = number of books in the 2nd stack, since there are twice as many books here.2N+30 = add on 30 to the previous amount to get the number of books in the first stack

In other words,

1st stack = 2N+302nd stack = 2N3rd stack = N

N is some positive whole number.

Add up all those expressions and set the sum equal to the 150 books total. Solve for N.

(1st stack) + (2nd stack) + (3rd stack) = total

(2N+30) + (2N) + (N) = 150

(2N+2N+N) + 30 = 150

5N + 30 = 150

5N = 150-30

5N = 120

N = 120/5

N = 24 which is the number of books in the 3rd stack

2N = 2*24 = 48 is the number of books in the 2nd stack.

2N+30 = 2*24+30 = 48+30 = 78 books in the 1st stack

Check:

1st+2nd+3rd = 78+48+24 = 150

The answer is confirmed.

Write seven tenths as a decimal

Answers

Answer:

seventh tenths as a decimal are 0.7

what’s the better deal? pls help

Answers

Answer:

the equations they're equal

Step-by-step explanation:

3.75 divided by 5 = 0.75

7.50 divided by 10 = 0.75

12.00 divided by 16 = 0.75

3. a Paul is thinking of a number that is greater than 10 and less than 20. What is the probability
that the number is not a prime number? Express your answer as a fraction only.

Answers

10/100, 10 numbers are prime and within 10 and 20 there are 100 #’s that are fractions

The probability that the number is not a prime number is 5/9.

What is Probability?

Probability refers to potential. A random event's occurrence is the subject of this area of mathematics.

The range of the value is 0 to 1. Mathematics has incorporated probability to forecast the likelihood of various events.

The degree to which something is likely to happen is basically what probability means.

Given:

Paul is thinking of a number that is greater than 10 and less than 20.

So, the prime number between 10 and 20 are 11, 13, 17, 19.

So, the probability that the number is not a prime number

= ( 9- 4)/ 9

= 5/9

Hence, the probability that the number is not a prime number is 5/9.

Learn more about Probability here:

https://brainly.com/question/11234923

#SPJ2

The hypotenuse of a right triangle measures 2 square root 15 centimeters and its shorter leg measures 2 square root 6
centimeters.

What is the measure of the larger acute angle of the triangle? Round your answer to the nearest
tenth of a degree.

Answers

Answer:

  50.8°

Step-by-step explanation:

The mnemonic SOH CAH TOA is intended to remind you of the relationship between side lengths and trig functions for a right triangle. Here, you are given the short side and the hypotenuse, and asked for the largest acute angle.

__

The short side is adjacent to the largest acute angle, so the relevant relation is ...

  Cos = Adjacent/Hypotenuse

  cos(β) = (2√6)/(2√15) = √0.4

The angle measure is found using the inverse cosine function:

  β = arccos(√0.4) ≈ 50.8°

The larger acute angle is about 50.8°.

Other Questions
Wary of the weather and steering by a star Write a balanced half-reaction for the reduction of permanganate ion mno4 to solid manganese dioxide mno2 in acidic aqueous solution. Be sure to add physical state symbols where appropriate Please help me will mark brainleist Only answer if you know, thank you! The publication of unsafe at any speed resulted in the widespread use of which feature on automobiles?. Question:Crash attenuatorsOptions:breakaway sign postmedianbarrierliquid or sand filled drum Lot's wife is one of many biblical references Rowlandson makes in her account. Research what Rowlandson means by saying, "and I understood something of Lot's wife's temptation." Why would she choose this reference to make an allusion? ayuda con esto ;v9 6- = -6 X Which expression is equivalent to P/6 +(q+8) After researching this topic, prepare a chart on a separate sheet of paper, of the following statistics for the nations of Kenya, South Africa, Zimbabwe, Ethiopia and Uganda. Statistics to be charted: People living with HIV/AIDS Children (ages 0-14) with HIV/AIDS Orphans due to AIDS Total population Then write a report in the space provided below, on what conclusions can be made from the facts about HIV/AIDS in Sub Sahara Africa. help a bro in need Use the method of symmetry to find the extreme value of each quadratic function and the value of x for which it occurs.f(x)=(x-3)(x+8a) x-intercepts are _________b) midpoint of the x-intercepts is ___________c) the extreme value is _______________d) f(_) = ______________ If f(x) = 2x2 1, what is f(x) when x = 3? 1 7 13 19 32.- There's going to be a new department at work. They've asked me toit up.O takesetO putObring 63. 26 Which expressions are equal to 23 ? 123 126 26 . 33 23 33 You are caring for a patient and he has not made any urine for the last 8 hours. What should you do? Help me find the figure , area formula and the area Translate the sentence into an equation.Nine less then the product of 5 and a number equals 7.Use the variable x for the unknown number. Which statement describes a change that occurred in Arkansas politics during the 1960s and 1970s? Question 12 options: Arkansas women began playing an active role politically. Politicians began pointing out the personal flaws of their opponents. Spending large amounts on campaign advertising was prohibited. The number of African American voters increased significantly. Which of the following is true of meiosis?Choose 1 answer:It occurs in all cells.It contains only one cell division.It does not require interphase to occur first.It produces haploid cells. 50 POINTS!!What is the name for the space inside the ribs that the lungs fill?cavitychestlungsdiaphragm Which statement most accurately states the claim in this passage? citizens of missouri cannot sue in a court of the united states. dred scott cannot sue in court because he is not a citizen. sanford should not have won the previous case in circuit court. the constitutions definition of citizenship does not include missouri.